Help I’m stuck!!!! Solve the equation below for x. Enter your answer as a fraction in lowes
terms, using the slash mark (/) as the fraction bar. 4/21 + 2/9
X=

Help Im Stuck!!!! Solve The Equation Below For X. Enter Your Answer As A Fraction In Lowesterms, Using

Answers

Answer 1

Answer:

26/63

Step-by-step explanation:

Try to make the denominators equal so that you can add them:

4(3)/21(3)= 12/63

2(7)/9(7)= 14/63

And now you add, which will give you:

12/63 + 14/63= 26/63


Related Questions

Which fraction converts to a repeating decimal number?
CA.
1
12
B.
718
C.
127
27
D.
E.
6
10
Reset

Answers

Answer: A.

Step-by-step explanation:

Data: Fraction that turning into a repeating decimal number=x

Only step: Divide all the fractions, 1/12, 7/8, 14/25, 17/20, 6/10

Explanation: The only way to find which fraction turns into a repeating decimal is by dividing all the fractions, this can be done in any order but for this problem, lets start with 1/12 which, when divided, turns into 0.083... which is a repeating decimal

With that being said, the answer would be A.(1/12)

I hope this helps(Mark brainliest if you'd like to)

in an isosceles triangle KLM, ∠K is congruent to ∠M and m ∠M=55°. find the m ∠L

Answers

Answer:

70°

Step-by-step explanation:

m ∠L = 180-(55+55)

= 180-110

=70°

i have no idea what to do can someone pls help me quick

Answers

The correct answer is D. I found this by plugging in zero for X into each equation until I got 1/4 for a Y
The correct answer is d

Please help I’ll give brainliest

Answers

Answer:

2 is the base

Step-by-step explanation:

2^3

2 is the base and 3 is the exponent

The answer is the base is 2 (first option)

Hi can anyone help me with this Q ? It’s for my final. I would appreciate it a lot !!
-
The angle of elevation from a boat to the top of a 125ft hotel is 22°
How far is the boat from the base of the hotel ? ( round to the nearest TENTH )

Answers

Answer: B

Step-by-step explanation:

Bc said so

3, 6, 9, 12, 15,
Tlie common difference " is

Answers

Answer:

The common difference is definitely 3. The common difference is the difference between the numbers in a sequence. In the sequence 3,6,9,12, and 15 the common difference is 3.

Put these numbers in order from least to greatest.

3/4,0.6, and 0.7

Answers

Answer:

O.6, 0.7, 3/4 (0.75)

Students are asked to estimate the number of gumballs in a jar. Sam says there are 228 gumballs. In actuality, there are 240 gumballs. What is the percent error

Answers

Answer:

5%

Step-by-step explanation:

Percent error = (actual - estimated) / actual x 100

(240 - 228) / 240 x 100 = 5%

Neil is buying steak for a cookout on Saturday. Steak is on sale for $9.62 per pound. If he buys 7.5 pounds of steak, how much money does he spend?

Answers

Answer:

$72.15

Step-by-step explanation:

x= pounds of steak

$[tex]9.62x[/tex] = money spent on steaks

If Neil buys 7.5 pounds of steak then x =7.5

$9.62(7.5) = $72.15

Answer:

$72.15

Step-by-step explanation:

1 pound = $9.62

7.5 pounds = $9.62 × 7.5

= $72.15

Pls help will mark brainliest and 30 points

Answers

1. Answer is option 3

2. Answer is option 3

Answer:

for #1 it's the 3rd one and for #2 it's the 4th one (2.8 meters)

if 2x + 3y = 12 and xy = 6, find the value of 8x^3 + 27y^3​

Answers

Answer:

The value of [tex]8\cdot x^{3} + 27\cdot y^{3}[/tex] is 432.

Step-by-step explanation:

Let be the following system of equations:

[tex]2\cdot x + 3\cdot y = 12[/tex] (1)

[tex]x\cdot y = 6[/tex] (2)

Then, we solve both for [tex]x[/tex] and [tex]y[/tex]:

From (1):

[tex]2\cdot x + 3\cdot y = 12[/tex]

[tex]2\cdot x = 12- 3\cdot y[/tex]

[tex]x = 6 - \frac{3}{2}\cdot y[/tex]

(1) in (2):

[tex]\left(6-\frac{3}{2}\cdot y \right)\cdot y = 6[/tex]

[tex]6\cdot y-\frac{3}{2}\cdot y^{2} = 6[/tex]

[tex]\frac{3}{2}\cdot y^{2}-6\cdot y + 6 = 0[/tex]

The roots of the polynomial are determined by the Quadratic Formula:

[tex]y_{1} = y_{2} = 2[/tex]

By (1):

[tex]x = 6 - \frac{3}{2}\cdot (2)[/tex]

[tex]x = 3[/tex]

If we know that [tex]x = 3[/tex] and [tex]y = 2[/tex], then the final value is:

[tex]z = 8\cdot x^{3}+27\cdot y^{3}[/tex]

[tex]z = 8\cdot 3^{3}+27\cdot 2^{3}[/tex]

[tex]z = 432[/tex]

The value of [tex]8\cdot x^{3} + 27\cdot y^{3}[/tex] is 432.

(a+b)(a-b)=? Đây là hằng đẳng thức nào?

Answers

Answer:

không

Step-by-step explanation:

Answer:

(a + b)(a - b) = a² - b²

Step-by-step explanation:

(a + b)(a - b) = a (a - b ) + b (a - b )

                 = a²  - ab + ba  - b²             [ ab = ba ]

                 = a² - ab + ab - b²               [ ab - ab = 0 ]

                 = a² - b²

Simplify the expression....​

Answers

Answer:

−3x^2+2x /x−2

Step-by-step explanation:

4x−9x^3/ 3x^2−4x−4

=  −9x^3+4x /3x^2−4x−4

=  x(−3x+2)(3x+2) /(3x+2)(x−2)

=  −3x^2+2x /x−2

Find the volume of the solid. Round your answer to the nearest hundredth.

Answers

Answer: 27.6

Step-by-step explanation:

tan 36 = 1/x

x =1/ tan 36 x = 1.38 which is your apothem

area of pentagon = 5x2x1.38/2 = 6.9

now do 6.9 x 4 = 27.6

The volume of the solid is 27.52 cubic yards

What is Three dimensional shape?

a three dimensional shape can be defined as a solid figure or an object or shape that has three dimensions—length, width, and height.

The given figure is a pentagonal prism

The base length of prism is 2yd

The height of prism is 4 yd

Volume of pentagonal prism V = 1/4 √(5(5+2√5))a²h

a is the base length and h is height

V = 1/4 √(5(5+2√5))4×4

V=27.52

Hence, the volume of the solid is 27.52 cubic yards

To learn more on Three dimensional figure click:

https://brainly.com/question/2400003

#SPJ2

Can someone helpppppppp

Answers

Answer:

f(-2) = -3

Step-by-step explanation:

When x = -2 we want the value of y

We use the closed circle value ( open circle does not have a value)

f(-2) = -3

Please help don’t understand at all.

Answers

Answer:

tringlae mixed witha football goal take the fool goal out and go from there

Step-by-step explanation:

give your answer in detail plzz ​

Answers

Answer:

60% did not germinate.

Step-by-step explanation:

100% = 5

how many percent are 2 of of these 5 ?

2 = 2/5 × 5 = 2/5 × 100% = 40%

all the other seeds are then the rest of 100%.

100% - 40% = 60%

=>

60% of the seeds did not germinate.

but 40% did.

In this addition problem, each letter stands for a different digit.
A
A
+ B B
-------------------
C C C

Find the value of A + B + C?

Answers

The answer is a+b+c equals abc

joey is going shopping for a new pair of sneakers. He finds a pair that have an original price of $155. They are on sale today for 30% off. How much does Joey pay for the sneakers including 8% sales tax?

Answers

Answer:, Joey will pay $117.18 for sneakers.

Step-by-step explanation:

Given: original price = $155

Discount rate = 30%

Tax rate = 8%

Price after discount = Original price - (Discount) x (original price)

[tex]= 155-0.30\times 155\\\\=155-46.5\\\\=\$\ 108.5[/tex]

Tax = Tax rate x (Price after discount)

[tex]= 0.08 \times 108.5[/tex]

= $ 8.68

Final price for sneakers = Price after discount + Tax

= $ (108.5+8.68)

= $ 117.18

Hence

What is the range of the given function?

{(–2, 0), (–4, –3), (2, –9), (0, 5), (–5, 7)}

A {x | x = –5, –4, –2, 0, 2}
B {y | y = –9, –3, 0, 5, 7}
C {x | x = –9, –5, –4, –3, –2, 0, 2, 5, 7}
D {y | y = –9, –5, –4, –3, –2, 0, 2, 5, 7}

Answers

[tex]\underline \bold{ \: \: \: \: \: \: \: \: \: \: \: \: \: \: \: \: \: \: \: \: \: \: \: \: \: \: \: \: \: \: \: \: \: \: \: \: \: \: \: \: \: \: \: \: \: \: \: \: \: \: \: \: \: \: \: \: \: \: \: \: \: \: \: \: \: \: \: \: \: \: \: \: \: \: \: \: \: \: \: \: \: \: }[/tex]

[tex]\huge\underline{\sf{\red{Problem:}}}[/tex]

What is the range of the given function?(–2, 0), (–4, –3), (2, –9), (0, 5), (–5, 7)

[tex]\huge\underline{\sf{\red{Choices:}}}[/tex]

A {x | x = –5, –4, –2, 0, 2}

B {y | y = –9, –3, 0, 5, 7}

C {x | x = –9,–5,–4,–3,–2,0,2,5,7}

D {y | y = –9,–5,–4,–3,–2,0,2,5,7}

[tex]\huge\underline{\sf{\red{Answer:}}}[/tex]

[tex] \quad \quad \underline{ \boxed{\sf{ \red{ B.)\: {y | y = –9, –3, 0, 5, 7} }} }}[/tex]

What is range of a function?The range of a function is the complete set of all possible resulting values of the dependent variable (y, usually), after we have substituted the domain.

The definition means:

The range is the resulting y-values we get after substituting all the possible x-values.

How to find the range?

The range of a function is the spread of possible y-values (minimum y-value to maximum y-value)Substitute different x-values into the expression for y to see what is happening. (Ask yourself: Is y always positive? Always negative? Or maybe not equal to certain values?)Make sure you look for minimum and maximum values of y.

[tex]\underline \bold{ \: \: \: \: \: \: \: \: \: \: \: \: \: \: \: \: \: \: \: \: \: \: \: \: \: \: \: \: \: \: \: \: \: \: \: \: \: \: \: \: \: \: \: \: \: \: \: \: \: \: \: \: \: \: \: \: \: \: \: \: \: \: \: \: \: \: \: \: \: \: \: \: \: \: \: \: \: \: \: \: \: \: }[/tex]

#CarryOnLearning

[tex]\sf{\red{✍︎ C.Rose❀}}[/tex]

Pls help. It’s quite late <3

Answers

Answer:

a) 120

Step-by-step explanation:

B is twice the size of A

15
9
determine the value
coso

Answers

Answer:

36.87°

Step-by-step explanation:

Given the right angle triangle :

To obtain the value of Cosθ ; we use the trigonometric relation :

Cosθ = Adjacent / Hypotenus

The adjacent angle isn't given :

Opposite = 9 ; hypotenus = 15

Adjacent = √(hypotenus ² + opposite ²)

Adjacent = √(15² - 9²)

Adjacent = √(225 -81)

Adjacent = √144 = 12

Hence,

Cos θ = 12/15

θ = Cos^-1(12/15)

θ = 36.87°

find the volume of this cone. round to the nearest tenth. l=10 r-6

Answers

Answer:

376.8

Step-by-step explanation:

Formula Volume Cone = πr² * h/3

Volume = 3.14 * 36 * 3 1/3

Volume ≈ 376.8

If my answer is incorrect, pls correct me!

If you like my answer and explanation, mark me as brainliest!

-Chetan K

Answer: 301.6

-------------------

answer above isn't right this is, trust me.

HELP!!!
f(x) =x^2. What is g(x)?

Answers

Answer:

C. g(x) = 4x²

Step-by-step explanation:

we have one specific point on g(x) : (1,4)

so, for x=1 the functional value (y) has to be 4.

A. (4×1)² = 4² = 16 and not 4. => wrong

B. 1/4 × 1² = 1/4 and not 4. => wrong

C. 4×1² = 4 and that IS 4 => correct

D. 16×1² = 16 and not 4 => wrong

The Jenkins family's monthly budget is
shown in the circle graph. The family has a
monthly income of $4,800. How much
money do they spend on transportation
each month?
Emergency fund
5%
Transportation
5%
Housing
30%
Medical
22%
Food
15%
Clothing
6%
Entertainment
7%
Savings
10%​

Answers

Answer:

$240

Step-by-step explanation:

They spend 5% of their monthly income on transportation, so you want to work out 5% of 4800.

An easy trick with 5% is to divide by 10 (drop the ending 0) and then divide by 2. This would be 4800/10/2, or 480/2, which gives 240.

**This content involves using percentages which you may wish to revise.

Please hurry i want the answer of this question please

Answers

[tex]\displaystyle\bf 1200=12*100=3*4*(2*5)^2=3*2^2*2^2*5^2=2^4*3^1*5^2 \\\\Answer: \boxed{ A)\quad a=4 \quad ; \quad b=1 \quad ; \quad c=2}[/tex]

Which verbal expression shows 4 ÷ n?

Your answer:

n divided by 4


the quotient of 4 and n


the product of 4 and n


4 decreased by n

Answers

Answer:

'the quotient of 4 and n ' is the correct answer

Can somebody help plz help me with this?

Answers

Answer:

N-8

Step-by-step explanation:

If the first three Fibonacci numbers are given as x, = 1, X2 = 1 and x3 = 2,
what is the value of n for which X, + Xp = 55?

Answers

Can you please help me with this question

Answer:

n=8

Step-by-step explanation:

Multiply the polynomials.
(8x2 + 6x + 8)(6x-5)
A. 48x3 - 4x2 + 18x + 40
B. 48x3 - 76x2 + 18% - 40
C. 48x3 - 4x2 + 78x - 40
D. 48x3 - 4x2 + 18x- 40

Answers

I’m pretty sure that the answer is D
Other Questions
evaluate g(x)=x/x-3, if g(1/2) Look up lamentings in the dictionary. What did Lennox hear in the night? What is the human female reproductive cycle regulated by? 1) What is the control group? 2) What is the experimental group? 3) What is the independent (manipulated) variable? 4) What is the dependent (responding) variable? 5) What should Tina's conclusion be? Using the following image, complete the statement below. I got all the answers I need the ones that is blank A 21-year-old woman presents with a 3-month history of a black mole on her right calf. She tells you that the lesion is enlarging and expanding. It began to itch about 3 weeks ago, and it has bled 2 times. She thinks that there may have been a mole near the same spot previously, but she is not certain. Her general health is good; there is no history of chronic illness, hospitalizations, or surgeries. She works as a professional model for a large advertisement agency. She does not take any prescription medication; she does not use tobacco, alcohol, or recreational drugs. Although she has dark hair, she has a fair skin, and she says that she usually burns with even short sun exposure. She does occasionally use a tanning booth prior to modeling events and vacations. There is no family history of skin cancer.VS stable, she looks anxious, but she is otherwise well.There is a dark brown-black nodule on the right calf 1 cm in diameter.On the surface of the nodule, there is a tiny area of crusting. There are no hairs. The nodule is asymmetrical, and its border is sharply demarcated; the color is uniform, and the elevation is regular. There is a narrow (1-2 mm) rim of erythema around most of the nodule. She has a sprinkling (about 25-30 in all) of melanocytic nevi on her trunk and legs. There is no significant local or distal lymphadenopathy. The liver is not palpable. The remainder of the physical examination is unremarkable.What is the most likely diagnosis? Given the equation 4square root of x minus 3 = -12, solve for x and identify if it is an extraneous solution.A. x = 0, solution is extraneousB. x = 0, solution is not extraneousC. x = 12, solution is extraneousD. x = 12, solution is not extraneous Which expression is equivalent to 10k + 17 - 7j - 18 - 11k?-8jk - 1-7j - k - 1-7j + k + 1-8j - kPLEASE HURRY !!!!20 POINTS!!! Answer this fast and correct and you will get a thanks, a brainliest and a 5-star review along with 10 points! The exponent on b when b^3 is multiplied by b^3 isA. 3B. 6C. 9 Hi Everyone hope u all r doin well Pls Answer What happens when dilute sulphuric acid is poured on a copper plate ? Indicate the equation of the given line in standard form, writing the answer in the equation box below.The line containing the longer diagonal of a quadrilateral whose vertices are A(2, 2), B(-2,-2), C(1, -1), and D16, 4). A balloon is filled with 80 liters of gas on a day where the temperature was 34 degrees at sea level which is 101.3 kPa and released. As the balloon rises to a certain altitude, the temperature drops to 0 degrees celsius and the balloon doubles in volume. What is the atmospheric pressure at that altitude? A crucible (container) of molten metal has an open top with an area of 5.000 m^2. The molten metal acts as a blackbody radiator. The intensity spectrum of its radiation peaks at a wavelength of 320 nm. What is the temperature of that blackbody? Find the distance between the pair of points: (0,-8)and (0,4) One of the biggest challenges that we all face, at least inmy opinion it's a challenge, is making difficult decisions,life-altering decisions. These decisions are not onlydifficult to make, but they can also bring consequencesthat are not easy to live with. However, if decisions are wellthought out and carefully considered they will be easier tomake and easier to live with.What feedback would be most helpful feedback to give the writer of thisparagraph?O A. Make the claim more personal.B. Address a counterclaim.O C. Revise the claim to make it clearer.D. Improve the spelling and grammar. Your English teacher asks you to write a story. You have to start with the story: I had a natural occurrence when I turned on the television. a sphere of mass 5kg and volume 210-5completely immersed in water find the buoyant force exerted water PLEASE HELP WILL MARK BRAINLIEST - An airplane travels 150 miles horizontally during a decrease of 35,000 feet vertically.1. What is the angle of descent?2. How long is the plane's path? What are the rational roots of f(d) = 5d - 6 + d-8?